Lösungen 1+2

This commit is contained in:
WieErWill 2022-03-26 17:27:58 +01:00
parent 2ebe1cdb48
commit c8714b337d
2 changed files with 39 additions and 2 deletions

Binary file not shown.

View File

@ -26,6 +26,10 @@
\SolutionEmphasis{\small}
\geometry{top=1cm,left=1cm,right=1cm,bottom=1cm}
\usepackage{pifont}
\newcommand{\cmark}{\ding{51}}
\newcommand{\xmark}{\ding{55}}
\pdfinfo{
/Title (Logik und Logikprogrammierung - Prüfungsvorbereitung)
/Creator (TeX)
@ -98,22 +102,33 @@
\begin{parts}
\part Der Korrektheitssatz der Aussagenlogik für den Wahrheitswertebereich $B$ lautet...
\begin{solution}
Für jede Menge von Formeln $\Gamma$ und jede Formel $\varphi$ gilt $\Gamma\vdash\varphi\Rightarrow\Gamma\vdash_B\varphi$.
\end{solution}
\part Eine Menge von Formeln $\Gamma$ heißt erfüllbar, wenn...
\begin{solution}
Sei $\Gamma$ eine Menge von Formeln. $\Gamma$ heißt erfüllbar, wenn es eine passende B-Belegung $B$ gibt mit $B(\gamma) = 1_B$ für alle $\gamma\in\Gamma$.
\end{solution}
\part Der Satz von Cook lautet...
\begin{solution}
Die Erfüllbarkeit einer endlichen Menge $\Gamma$ ist NP-vollständig.
\end{solution}
\part Zwei Formeln $\alpha$ und $\beta$ heißen äquivalent, wenn...
\begin{solution}
Zwei Formeln $\alpha$ und $\beta$ heißen äquivalent $(\alpha\equiv\beta)$, wenn für alle passenden B-Belegungen $B$ gilt: $B(\alpha) =B(\beta)$.
\end{solution}
\part Der Kompaktheitssatz der Aussagenlogik lautet...
\begin{solution}
Sei $\Gamma$ eine u.U. unendliche Menge von Formeln. Dann gilt $\Gamma$ unerfüllbar $\Leftarrow\Rightarrow\exists\Gamma'\subseteq\Gamma$ endlich: $\Gamma'$ unerfüllbar
\end{solution}
\part Eine Horn Klausel ist eine Formel der Form
\begin{solution}
Eine Hornklausel hat die Form $(\lnot\bot\wedge p_1\wedge p_2\wedge ... \wedge p_n)\rightarrow q$ für $n\geq 0$, atomare Formeln $p_1 ,p_2 ,... ,p_n$ und $q$ atomare Formel oder $q=\bot$.
Eine Hornformel ist eine Konjunktion von Hornklauseln.
\end{solution}
\end{parts}
@ -121,14 +136,36 @@
\begin{parts}
\part Werte die Formel $\varpi_a=\lnot p \wedge \lnot\lnot p$ im Heytingschen Wahrheitswertebereich $H_{\mathbb{R}}$ aus für die $H_{\mathbb{R}}$-Belegung $B$ mit $B(p)=\mathbb{R}\backslash \{0\}$
\begin{solution}
$B_{H_{mathbb{R}}}(\lnot p \wedge \lnot\lnot p)= Inneres(\mathbb{R}/ p)\cap p= 1$
\end{solution}
\part Überprüfe ob die Formel $\varphi_B=(\lnot p\rightarrow \lnot p)\rightarrow p$ eine $K_3$-Tautologie ist. Ist $\varphi_b$ eine $B_{\mathbb{R}}$ Tautologie?
\begin{solution}
\begin{tabular}{c|c|c|c}
$p$ & $\lnot p$ & $\phi=(\lnot p\rightarrow \lnot p)$ & $\phi\rightarrow p$ \\\hline
0 & 1 & 1 & 0 \\
$\frac{1}{2}$ & $\frac{1}{2}$ & 1 & $\frac{1}{2}$ \\
1 & 0 & 1 & 1 \\
\end{tabular}
\end{solution}
\part Überprüfe ob die semantische Folgeung $\{p\rightarrow q, q\rightarrow r\}\Vdash_B r\rightarrow\lnot p$ gilt.
\begin{solution}
\begin{tabular}{c|c|c|c|c|c|c|c}
$p$ & $q$ & $r$ & $\lnot p$ & $\Gamma_1=p\rightarrow q$ & $\Gamma_2=q\rightarrow r$ & $\Phi=r\rightarrow\lnot p$ & $\Gamma\Vdash\Phi$ \\\hline
0 & 0 & 0 & 1 & 1 & 1 & 1 & \cmark \\
0 & 0 & 1 & 1 & 1 & 1 & 1 & \cmark \\
0 & 1 & 0 & 1 & 1 & 0 & 1 & \\
0 & 1 & 1 & 1 & 1 & 1 & 1 & \cmark \\
1 & 0 & 0 & 0 & 0 & 1 & 1 & \\
1 & 0 & 1 & 0 & 0 & 1 & 0 & \\
1 & 1 & 0 & 0 & 1 & 0 & 1 & \\
1 & 1 & 1 & 0 & 1 & 1 & 0 & \xmark
\end{tabular}
Folgerung gilt nicht
\end{solution}
\end{parts}